Lema (sub-base de Alexander)

Seja $(X,\tau)$ espaço topológico e $\mathcal{B}$ sub-base para $X$. Então $$(X,\tau)\text{ é compacto}\iff \text{toda cobertura }\mathcal{F}\subset \mathcal{B}\text{ de }X\text{ admite subcobertura finita}$$

Demonstração: $(\Rightarrow)$ Trivial, já que $\mathcal{B}\subset \tau$.

$(\Leftarrow)$ Suponha, por absurdo, $(X,\tau)$ não compacto e seja $\mathcal{C}$ a coleção de todas as coberturas abertas de $X$ que \textbf{não} admitem subcobertura finita. Dada uma cadeia $\{\mathcal{F}_{\alpha}\}_{\alpha \in I}\subset \mathcal{C}$, se $M:=\bigcup_{\alpha \in I} \mathcal{F}_{\alpha}$, então $$\mathcal{F}_{\alpha}\subset M\ \forall \alpha \in I$$ Além disso, $M\in \mathcal{C}$, pois $M$ é cobertura aberta de $X$, e se $M$ admitisse subcobertura finita $\{Y_1,\dots,Y_k\}$, então para cada $1\le i\le k$ existe $\alpha_i\in I$ tal que $Y_i\in \mathcal{F}_{\alpha_i}$. Sem perda de generalidade, podemos supor $$\mathcal{F}_{\alpha_1}\subset \mathcal{F}_{\alpha_2}\subset \dots \subset \mathcal{F}_{\alpha_k}$$ donde $\{Y_1,\dots,Y_k\}\subset \mathcal{F}_{\alpha_k}$, contradizendo $\mathcal{F}_{\alpha_k}$ não ter subcobertura finita.
Pelo Lema de Zorn, $\mathcal{C}$ admite um elemento maximal $C$, e como trabalhamos com a relação de inclusão, tal elemento é único. Note que $C\cap \mathcal{B}$ não é cobertura de $X$, pois caso conttrário, já que $C\cap \mathcal{B}\subset \mathcal{C}$, por hipótese $C\cap \mathcal{B}$ admite subcobertura finita, contrariando o fato de que $C$ não admite cobertura finita, pois $C\cap \mathcal{B}\subset C$. Assim, existe $x_0\in X$ com $$x_0\not \in B\ \forall B\in C\cap \mathcal{B}$$ e $x_0\in B_0$ para algum $B_0\in C$ ($C$ é cobertura de $X$), e como $B_0$ é aberto a família das interseções finitas dos elementos de $\mathcal{B}$ forma uma base, então existem $B_1,\dots,B_k\in \mathcal{B}$ tais que $$x_0\in \bigcup_{i=1}^k B_i\subset B_0$$ Como $x_0$ não pertence à união dos elementos de $C\cap \mathcal{B}$, então $B_i\not \in C\cap \mathcal{B},\ i=1,\dots,k$.
Observe que para cada $1\le i\le k$, $C\cup \{B_i\}$ é cobertura de $X$, e como $C$ é o maior elemento de $\mathcal{C}$, deve admitir subcobertura finita $\{B_i\}\cup C_i$, onde $C_i\subset C$.
Por fim, dado $x\in X$, então se $x\not \in B_0\Rightarrow x\not \in \bigcap_{i=1}^k B_i\Rightarrow x\not \in B_j\text{ para algum }1\le j\le k\Rightarrow x$ pertence a algum elemento de $C_j$, pois $\{B_j\}\cup C_j$ é cobertura de $X$. Logo, $\{B_0\}\cup \left(\bigcup_{i=1}^k C_i\right)\subset C$ é cobertura finita de $X$, $\textbf{absurdo}_{\ \blacksquare}$.